¿Se puede acelerar el punto de referencia en un marco inercial al aplicar la ecuación de par?

Torque sobre un punto = tasa de cambio del momento angular sobre ese punto.

Digamos que estamos en un marco inercial y vemos un cuerpo acelerando y rotando y otro punto (ya sea una parte de este cuerpo o un punto externo) acelerando.

¿Podemos aplicar T = dL/dt para ese cuerpo sobre ese punto de aceleración si estamos en un marco inercial?

Supongo que podemos, ya que el momento angular depende del punto de referencia. ¿Hay algún requisito para que este punto no sea acelerante?

Oh, eso fue un error tipográfico. Lo corregiré.

Respuestas (3)

La ley que estableces solo es válida para el centro de masa o para un punto fijo en el espacio.

La ley de rotación de Euler establece:

El momento de torsión neto de un objeto alrededor del centro de masa es igual a la tasa de cambio del momento angular medido en el centro de masa .

(1) T C = d d t ( L C ) = d d t ( I C ω ) = I C ω ˙ + ω × L C

donde el punto C denota el centro de masa. Este es un equivalente directo al hecho de que la fuerza neta sobre un cuerpo describe el movimiento del centro de masa solamente. El movimiento restante (sobre el centro de masa) está descrito por la ley de Euler.

El centro de masa puede estar acelerando (y por lo general lo está) ya que por lo general tanto el par como la fuerza se consideran al mismo tiempo.

(2) F = d d t ( pag ) = d d t ( metro v C ) = metro v ˙ C

Entonces, ¿qué sucede ahora en una ubicación A diferente ? Considere el vector de ubicación C del centro de masa, relativo a A

El momento angular en A es

(3) L A = L C + C × pag

El par neto en A es

(4) T A = T C + C × F

La derivada total del momento angular en A es

(5) d d t ( L A ) = d d t ( L C + C × pag ) = T C + d C d t × pag + C × d pag d t F = T A + ( v C v A ) × pag ( v C v A ) × ( metro v C ) = v A × metro v C = pag × v A

Produzco la siguiente ley (si nadie más la reclama, llámela ley ja72 ).

La tasa de cambio del momento angular en un punto arbitrario no fijo A es igual al par neto en A , más el producto cruzado del momento lineal con la velocidad de A.

(6) d d t ( L A ) = T A + pag × v A

Las condiciones en las que la derivada del momento angular es exactamente el momento de torsión neto en un punto _A_sobre un cuerpo rígido son las siguientes:

  1. El cuerpo experimenta una rotación pura con momento lineal cero, pag = 0
  2. El punto A está fijo en el espacio, o instantáneamente fijo, v A = 0
  3. El punto A está en el centro de masa, haciendo su movimiento paralelo al impulso, v A pag
  4. El punto A se encuentra en una línea paralela al eje de rotación, pero a través del centro de masa, v A v C
En la derivación original de Torque = dL/dt, en ninguna parte usamos esa suposición. phys.libretexts.org/TextMaps/Classical_Mechanics_TextMaps/…
@xasthor - ¿No es la forma correcta de la derivada del tiempo?
L ˙ = i ( r ˙ i × pag i + r i × pag ˙ i )
El producto cruzado es un operador lineal y la regla del producto regular se aplica a las derivadas.
@xasthor - también lo que es F i y F i j que su suma es igual a la velocidad?
@xasthor: las leyes de movimiento de rotación de Euler se aplican solo en el centro de masa. No puede elegir una ubicación arbitraria para el par neto ya que un cuerpo gira solo si la línea de acción de una fuerza no está en el centro de masa. Además, el momento angular que no está en el centro de masa incluye el momento del componente de momento que es una función del movimiento lineal. Consulte La derivación de ecuaciones de movimiento no en el centro de masa para ver por qué d d t ( L A ) T A en cualquier punto A.
derivado de r i es v i que al cruzarse con pag i =0 ya que están en la misma dirección. por lo tanto, la suma se simplifica en la suma de pares. nada sobre esa suma especifica que L o T se trata del centro de masa. parece tratarse de cualquier punto arbitrario
@xasthor entiendo esa parte, y la tengo en mi respuesta también como v C × pag = 0 , pero hay otros términos que no son cero que faltan en TextMaps. ¿Puedes explicar cómo? r ˙ i = F i + F i j ¿obras? ¿Cómo es la velocidad igual a la suma de las fuerzas? Se supone que el centro de masa está en el origen en esa publicación.
sí, esa ecuación no tiene sentido. No lo leí detenidamente antes de publicarlo, lo siento. consulte este en google.co.in/url?sa=t&source=web&rct=j&url=http://… página 13. Aquí, cito a Daniel Kleppner: "El momento de torsión externo sobre el punto S es igual a la derivada temporal del momento angular del sistema con respecto a ese punto"
Mientras leo esto, ¿puede mirar esta respuesta relacionada que establece el momento lineal y angular total para un sistema de partículas? Esta sería la base utilizada para mostrar que el par y la velocidad del momento angular solo se igualan en el centro de masa, o en un punto fijo o si el centro de masa está fijo. En los modelos de sistema de partículas, el punto de medición del momento angular es fijo (el origen) y así con v A = 0 entonces d d t ( L A ) = T A
@xasthor: para un punto (como en un solo punto), sí, es cierto porque la velocidad es paralela al impulso. Para un sistema de partículas (como un cuerpo rígido) con cualquier tipo de rotación, el momento no es paralelo a la velocidad de ningún punto arbitrario.
Daniel Kleppner lo demostró para un cuerpo rígido en la página 13 de ese documento.
@xasthor: el punto S está fijo en el espacio en la página 13. Puede ver que está implícito entre las ecuaciones (19.5.3)y (19.5.4). Si intentas pasar por la misma prueba con un punto S montado en el cuerpo y obtienes mi ley. La diferencia es que la ubicación de cada punto j es r j = r S + r j , S así que en(19.5.4) d r j d t = v S + v j , S o
d r j d t × pag j = v S × pag j 0
He agregado en mi respuesta las condiciones que harían el cero superior.
@xasthor: tome una rueda rodante libre y calcule el momento angular de un punto en el borde. Coloque un sistema de coordenadas en el punto de contacto y trace el punto alrededor. Ahora encuentre si se conserva el momento angular con respecto a ese punto. No lo es. Pero el momento de torsión neto sobre el punto es siempre cero.
@xasthor Hola. Perdona por molestarte, pero no entiendo cómo es cierta la ecuación 3.11.3 en el enlace que publicaste.
Esta publicación muestra que un concepto incorrecto conduce a una conclusión incorrecta. En la mecánica newtoniana, la relación d L d t = τ es válido para todo marco inercial. La conclusión aquí rompe este principio equivalente de la mecánica newtoniana.
Los conceptos erróneos son que (1) el autor cree que la relación entre el momento angular y el torque solo es válida en el marco COM; (2) el autor cree que la cantidad de movimiento de un cuerpo rígido siempre es igual a la cantidad de movimiento medida en el punto COM, lo cual no es cierto, en contra de la definición básica de cantidad de movimiento.
Explicación detallada, consulte la respuesta que publico a continuación.
@ytlu Las páginas 9-10 de esta conferencia establecen la ecuación (1) anterior. De acuerdo en la relación entre y. cantidad de movimiento y momento de torsión en el centro de masa de un cuerpo rígido y en el origen de un marco inercial?
@JohnAlexiou Las páginas 9 y 10 concluyeron que la tasa de cambio del momento angular es igual al par tanto para el origen O como para el origen C. No entiendo sus puntos.
Estoy de acuerdo con este punto ya que ambos puntos harían que mi término extra fuera cero. Tengo una presentación más detallada de mi argumento de por qué mi ecuación es correcta, incluido un ejemplo que produce el resultado correcto con mi ecuación y un resultado incorrecto con su ecuación. Todo en el contexto de un solo cuerpo rígido en movimiento visto por un solo observador inercial .
@ytlu: el mismo documento que el comentario anterior, pero publicado en HTML en lugar de PDF.
@JohnAlexiou Es lo mismo. No hay nada que haga correcto lo que dices. Periodo. Lo incorrecto no puede ser lo correcto.
@ytlu: ¿viste al final el ejemplo? Con los términos "extra" da la solución incorrecta. Después de un cuidadoso reexamen de todo, mantengo mis publicaciones. Recuerde, estoy hablando de un solo marco de referencia que observa múltiples puntos, y en los ejemplos que menciona, está hablando de dos marcos de referencia. Estoy hablando de un marco.
@JohnAlexiou "hablando de un solo marco de referencia que observa múltiples puntos", este dicho sin sentido. Un punto de observación es un marco de coordenadas. Aparte de eso, no hay puntos de observación. Este es un concepto muy básico de la física.
@ytlu: en física, la ubicación del observador es el marco de coordenadas. Donde mira el observador es un punto de observación. El marco de coordenadas define los vectores base y el origen donde se miden las distancias a los puntos de observación . Esto es fundamental para la física. En mi escenario. Estoy parado en el suelo sosteniendo un palo largo que apunta a diferentes lugares en el espacio. Estoy mirando el centro de masa C o algún otro punto arbitrario A observando lo que sucede en ese lugar.
@JohnAlexiou Cuando escribes L A , significa que un momento angular medido en el Marco A (aunque es un punto sin una cruz unida a él). Esto es sin ambigüedad. La definición es la medida del vector de posición de A × medida de velocidad de A.
@JohnAlexiou Ni siquiera sé cómo nombrar tu cantidad: vector de posición medido desde A × medida de velocidad de O. Podría ser cualquier cosa menos L A .
@ytlu- L A es el momento angular si el cuerpo gira alrededor de A se define exactamente como
L A = i r i × metro i v i
dónde r i es la ubicación de la partícula wrt point A . Está relacionado con el momento angular alrededor del centro de masa. L C con la relacion que estas disputando
L A = L C + r A C × pag
Esto también se encuentra en la literatura como la ecuación 19.5.19 aquí .
Tu error no está en la posición, sino en la velocidad. Su v i NO es la velocidad medida desde A. Por lo tanto, su L A no es el momento angular medido desde A.
En mi comentario anterior hice una simplificación que confunde las cosas. La definición de L A está basado en r i la ubicación de la partícula en relación con el observador, y v i la velocidad de la partícula (desde el punto de vista del observador) de la siguiente manera
L A = i ( r i r A ) × metro i v i
y ahora la velocidad es claramente solo v i = d d t r i

Esto no es una respuesta. Publico para aclarar los errores en la respuesta de John.

Ahora, centrémonos en tu respuesta anterior. Te muestro claramente los errores.

ecuación (3) en esta publicación:

Eq.(3) en la publicación de JohnEc. (3) en la publicación de John.

donde tu definido C como : Considere el vector de ubicación c del centro de masa, relativo a A . El punto A tiene una velocidad v A refiriéndose al origen O. Y pag = metro v C es la velocidad del centro de masa medida en el marco O.

Con estas definiciones, su ecuación (3) NO es correcta. Como A se mueve con velocidad v A , por lo tanto, en el marco A, el momento lineal total del cuerpo rígido es metro ( v C v A ) . Este error lo llevó a una conclusión incorrecta en la Ec. (5) y la Ec. (6).

Desde v C y pag se mide en el marco O y C se mide en el marco A. La ecuación. (3) debe escribirse como:

L A = L C + C × metro ( v C v A )

Con todas las cantidades medidas en el marco A. Esto corregirá sus resultados en Eq.(5) y Eq.(6), reders d L A d t = τ A .


Siguiendo a John, cambio algunas notaciones:

  • C: el marco del centro de masa
  • R : la posición del centro de masa en el marco A ( C de Juan.)
  • marco A: otro marco inercial, que tiene una velocidad constante relativa respecto al marco C.
  • v A : velocidad del punto medida en el marco A.
  • v C : velocidad del punto medida en el marco C.
  • r A : posición de la medida del punto en el cuadro A.
  • r C : posición de la medida del punto en el cuadro C.

La relación de posiciones:

(1) r A = R + r C .

Derivar la ecuación (1) conduce a la relación de velocidad:

(2) v A = V + v C .
dónde V es la velocidad de COM en el marco A. Debe ser una constante para que ambos marcos sean inerciales.

La definición de los momentos angulares:

L A = metro r A × v A L C = metro r C × v C

Su relación se puede encontrar a partir de la ecuación (1) y la ecuación (2):

L A = metro ( R + r C ) × ( V + v C ) = metro ( R + r C ) × V + metro R × v C + metro r C × v C = { L C + metro R × v C } + metro ( R + r C ) × V = { término de Juan } + { término perdido. }

El término dentro del corchete de curry se muestra en la ecuación de John. El último término se pierde de su relación de momento angular. Ciertamente así es metro r A × V , el momento extraagular de la partícula en el marco A debido al movimiento relativo entre el marco A y C.

La relación incorrecta en el momento angular lleva a su conclusión errónea. Su conclusión es una acusación muy seria contra el principio equivalente de todos los marcos inerciales, un concepto base muy importante de la mecánica newtoniana .

d L A d t = metro d r A d t × v A + metro r A × d v A d t = 0 + metro r A × d v A d t = metro r A × d d t ( V + v C ) = r A × d ( metro v C ) d t = r A × F = τ A .

Siempre que el movimiento relativo sea una velocidad constante d V d t = 0 , la tasa de cambio del momento angular es igual al par.

Describa un movimiento de muchos cuerpos en un cierto marco inercial:

L t o t a yo = i metro i r i × v i

Y el cambio de tarifa:

d L t o t a yo d t = i metro i d r i d t × v i + i r i × d pag i d t = i metro i 0 + i r i × F i = i τ i = τ t o t a yo

Si intenta argumentar que lejos del marco del centro de masa:

d r i d t × v i 0.

Tienes que hacerlo mucho mejor que un dicho de mano.

Lo ilustraré con un ejemplo simple, una mancuerna de dos puntos de masa puntual ( 2 k gramo ) separados por alambre sin masa (2m). Su masa central se mueve con velocidad V = 4 metro / s X + 3 metro / s y . En el sistema de masa central C, giran alrededor del centro con una frecuencia v = 1 / s .

Movimiento y rotación de una mancuerna.

Describe el movimiento de estas dos masas en el centro de masa (marco C) r C i = ( X i , y i ) para i = 1 , 2 :

X 1 = porque ( 2 π t ) ; y 1 = pecado ( 2 π t ) X 2 = porque ( 2 π t ) ; y 2 = pecado ( 2 π t ) Velocidad: v 1 X = 2 π pecado ( 2 π t ) ; v 1 y = 2 π porque ( 2 π t ) v 2 X = 2 π pecado ( 2 π t ) ; v 2 y = 2 π porque ( 2 π t )

El momento angular en C L C = ( 0 , 0 , L z ) = ( 0 , 0 , L 1 z + L 2 z ) :

L 1 z = 2 X 1 v 1 y 2 y 1 v 1 X = porque ( 2 π t ) 4 π porque ( 2 π t ) pecado ( 2 π t ) { 4 π pecado ( 2 π t ) } = 4 π . L 2 z = 4 π ,  similarmente
El momento angular total en el marco C es 8 π , una constante en el tiempo.

Ahora, examine el momento angular observado en el Cuadro A, r i A = ( X i , y i ) , y la posición de CM en el cuadro A R = ( 4 t , 3 t ) :

X 1 = 4 t + porque ( 2 π t ) ; y 1 = 3 t + pecado ( 2 π t ) X 2 = 4 t porque ( 2 π t ) ; y 2 = 3 t pecado ( 2 π t ) Velocidad: v 1 X = 4 2 π pecado ( 2 π t ) ; v 1 y = 3 + 2 π porque ( 2 π t ) v 2 X = 4 + 2 π pecado ( 2 π t ) ; v 2 y = 3 2 π porque ( 2 π t )

Ahora, verifique el momento angular en el marco A:

L 1 z = 2 { 4 t + porque ( 2 π t ) } { 3 + 2 π porque ( 2 π t ) } 2 { 3 t + pecado ( 2 π t ) } { 4 2 π pecado ( 2 π t ) } = 4 π + 6 porque ( 2 π t ) + dieciséis t π porque ( 2 π t ) 8 pecado ( 2 π t ) + 12 t π pecado ( 2 π t ) . L 2 z = 2 { 4 t porque ( 2 π t ) } { 3 2 π porque ( 2 π t ) } 2 { 3 t pecado ( 2 π t ) } { 4 + 2 π pecado ( 2 π t ) } = 4 π 6 porque ( 2 π t ) dieciséis t π porque ( 2 π t ) + 8 pecado ( 2 π t ) 12 t π pecado ( 2 π t ) .

Finalmente, el momento angular resultante en el Marco A:

L A z = L 1 z + L 2 z = 8 π .

También es una constante en el tiempo, aunque el marco A tiene un movimiento relativo con el marco C.

Compruebe la identidad de John para la masa 1:

R × metro v 1 = z ^ { dieciséis t π porque ( 2 π t ) + 12 t π pecado ( 2 π t ) } L 1 z + [ R × metro v 1 ] z = 4 π + dieciséis t π porque ( 2 π t ) + 12 t π pecado ( 2 π t ) L 1 z

Gracias por la explicación detallada. Estoy revisando ahora, y veo que hay algunas diferencias sutiles en la interpretación. Por ejemplo v C y v A for es la velocidad de un cuerpo rígido de cualquier partícula que se encuentre en las ubicaciones C y A respectivamente, ambas medidas desde el mismo marco inercial (el marco universal).
Donde muestro solo para una partícula. La descripción de muchas partículas agregará una suma sobre todas las partículas. Cada uno de ellos tiene una forma similar en la relación de momento entre dos marcos.
Agregué un ejemplo simple que continúa en mi publicación, toda la cantidad se puede verificar en el modelo simple.
¿Puede revisar esta publicación y ver si está de acuerdo? Esta es la base sobre la cual estoy construyendo mi refutación. El argumento desde el punto de vista de la suma de partículas. También esta publicación similar y también entra en las leyes de Newton.
En la primera publicación, no puedo encontrar los temas relacionados con su relación de momento angular que estaba mal.
Además, en la segunda publicación, no proporcionó ninguna derivación de la relación L a = L C + C × pag . ¿Puede aclarar su argumento señalando la razón por la cual esta ecuación es válida?
Como se comentó en otro lugar, lea esta publicación que se expande en detalles insoportables por qué las ecuaciones anteriores y mis ecuaciones siguen siendo válidas. Creo que nuestro desacuerdo está en la interpretación de lo que significa exactamente cada término, y es por eso que tengo muchos detalles incluidos en este documento.
Agregué mi comentario en una de sus publicaciones anteriores al comienzo de esta publicación. Su publicación obviamente tuvo un error similar en la transformación del momento angular. No mantuvo un sistema de notación claro y mezcló cantidades físicas de diferentes marcos dentro de una ecuación sin la corrección adecuada.
En la publicación que vinculó, la ecuación (3) y la ecuación (4) la ω debe ser diferente El ω en la ecuación (3) es la velocidad angular de rigit boday girando alrededor del centro de masa, y en la ecuación (4) la ω es la rotación de COM alrededor del punto A. No son en general un mismo ω . y de nuevo, en la ecuación (9) apareció el mismo error. El momento angular en A es L A = i metro i ( r i r a ) × ( v i v a ) . Este es tu error intrínseco.

Si el cuerpo está en movimiento sin restricciones, utilizando el centro de masa (CM) como punto de referencia, la suma de los pares de las fuerzas reales es igual al cambio en el momento angular, incluso si el CM está acelerando. Si el cuerpo está obligado a girar alrededor de un punto que no sea el CM y ese punto está acelerando, entonces se deben considerar fuerzas/torques ficticios utilizando ese punto como punto de referencia. Esto se discute con cierta extensión en el texto Mechanics, de Symon.